Математика ЕГЭ
Русский язык ЕГЭ
Математика 5-7
Математика ОГЭ
Информатика
Физика
Обществознание
Кликните, чтобы открыть меню

18. Задачи с параметром

1. Вспоминай формулы по каждой теме
2. Решай новые задачи каждый день
3. Вдумчиво разбирай решения

Задачи с параметром из ЕГЭ прошлых лет (страница 2)

Задание 8 #3267
Уровень задания: Равен ЕГЭ

Найдите все значения \(a\), при каждом из которых уравнение \[\sqrt{1-4x}\cdot \ln(9x^2-a^2)=\sqrt{1-4x}\cdot \ln (3x+a)\]

имеет ровно один корень.

 

(ЕГЭ 2017, основная волна)

Данное уравнение можно переписать как \[\begin{cases} \sqrt{1-4x}\cdot \ln \dfrac{(3x-a)(3x+a)}{3x+a}=0\\[2ex] 3x+a>0\end{cases} \quad\Leftrightarrow\quad \begin{cases} \sqrt{1-4x}\cdot \ln (3x-a)=0\\ 3x+a>0\end{cases}\] Система имеет два корня:
1) \(x_1=\frac14\), если он удовлетворяет \(3x+a>0\) и \(3x-a>0\): \[\begin{cases} \dfrac34+a>0\\[1ex] \dfrac34-a>0\end{cases} \quad\Leftrightarrow\quad -\dfrac34<a<\dfrac34\]
2) \(x_2=\frac{a+1}3\), если он удовлетворяет \(3x+a>0\) и \(1-4x\geqslant 0\): \[\begin{cases} a+1+a>0\\[1ex] 1-\dfrac43a-\dfrac43\geqslant 0\end{cases}\quad\Leftrightarrow\quad -\dfrac12<a\leqslant -\dfrac14\]

 

Рассмотрим случаи, когда данная система имеет ровно один корень. Пусть \(x_1\ne x_2\), то есть \(a\ne -\frac14\).
1. Пусть \(x_1=\frac14\) – единственное решение системы.
\(x_1\) будет корнем, если \(-\frac34<a<\frac34\), \(x_2\) не будет корнем, если \(a\in \left(-\infty;-\frac12\right]\cup\left(-\frac14;+\infty\right)\). Пересекая эти значения, а также учитывая, что \(a\ne -\frac14\), получаем: \[a\in \left(-\dfrac34;-\dfrac12\right]\cup\left(-\dfrac14;\dfrac34\right)\] 2. Пусть \(x_2=\frac{a+1}3\) – единственное решение системы.
\(x_1\) не будет корнем, если \(a\in \left(-\infty;-\frac34\right]\cup\left[\frac34;+\infty\right)\), \(x_2\) будет корнем, если \(-\frac12<a\leqslant -\frac14\). Пересекая эти значения, а также учитывая, что \(a\ne -\frac14\), получаем: \[a\in \varnothing\]

Пусть \(x_1=x_2\). Тогда \(a=-\frac14\). Заметим, что при этом значении что \(x_1\), что \(x_2\) являются решением, следовательно, оно нам подходит.
Итоговый ответ: \[a\in \left(-\dfrac34;-\dfrac12\right]\cup\left[-\dfrac14;\dfrac34\right)\]

Ответ:

\(a\in \left(-\dfrac34;-\dfrac12\right]\cup\left[-\dfrac14;\dfrac34\right) \)

Задание 9 #3219
Уровень задания: Равен ЕГЭ

Найдите все значения параметра \(a\), при каждом из которых уравнение \[\sqrt{5x-3}\cdot \ln (x^2-6x+10-a^2)=0\]

имеет ровно один корень на отрезке \([0;3]\).

 

(ЕГЭ 2017, основная волна)

ОДЗ уравнения: \[\begin{cases} 5x-3\geqslant 0\quad (1)\\ x^2-6x+10-a^2>0 \quad (2)\end{cases}\] Корнями уравнения будут \[\left[ \begin{gathered}\begin{aligned} &5x-3=0\\ &x^2-6x+10-a^2=1 \end{aligned}\end{gathered}\right. \quad\Rightarrow\quad \left[ \begin{gathered}\begin{aligned} &x_1=\dfrac35\\[2ex] &(x-3)^2=a^2 \end{aligned}\end{gathered}\right. \quad\Rightarrow\quad \left[ \begin{gathered}\begin{aligned} &x_1=\dfrac35\\[2ex] &x_2=3+a\\ &x_3=3-a \end{aligned}\end{gathered}\right.\] Заметим, что корень \(x_1\) удовлетворяет \((1)\), корни \(x_2\) и \(x_3\) удовлетворяют \((2)\). Также заметим, что корень \(x_1\) принадлежит отрезку \([0;3]\).
Рассмотрим три случая:

 

1) \(a>0\). Тогда \(x_2>3\), \(x_3<3\), следовательно, \(x_2\notin [0;3].\) Тогда уравнение будет иметь один корень на \([0;3]\) в одном из двух случаях:
\(x_1\) удовлетворяет \((2)\), \(x_3\) не удовлетворяет \((1)\), или совпадает с \(x_1\), или удовлетворяет \((1)\), но не входит в отрезок \([0;3]\) (то есть меньше \(0\));
\(x_1\) не удовлетворяет \((2)\), \(x_3\) удовлетворяет \((1)\) и не равен \(x_1\).
Заметим, что \(x_3\) не может быть одновременно меньше нуля и удовлетворять \((1)\) (то есть быть больше \(\frac35\)). Учитывая это замечание, случаи записываются в следующую совокупность: \[\left[ \begin{gathered}\begin{aligned} &\begin{cases} \dfrac9{25}-6\cdot \dfrac35+10-a^2>0\\[2ex] 3-a\leqslant \dfrac35\end{cases}\\ &\begin{cases} \dfrac9{25}-6\cdot \dfrac35+10-a^2\leqslant 0\\[2ex] 3-a> \dfrac35\end{cases} \end{aligned}\end{gathered}\right.\] Решая данную совокупность и учитывая, что \(a>0\), получим: \[a\in \left[\dfrac{12}5;\dfrac{13}5\right)\]

2) \(a=0\). Тогда \(x_2=x_3=3\in [0;3].\) Заметим, что в этом случае \(x_1\) удовлетворяет \((2)\) и \(x_2=3\) удовлетворяет \((1)\), то есть уравнение имеет два корня на \([0;3]\). Это значение \(a\) нам не подходит.

 

3) \(a<0\). Тогда \(x_2<3\), \(x_3>3\) и \(x_3\notin [0;3]\). Рассуждая аналогично пункту 1), нужно решить совокупность: \[\left[ \begin{gathered}\begin{aligned} &\begin{cases} \dfrac9{25}-6\cdot \dfrac35+10-a^2>0\\[2ex] 3+a\leqslant \dfrac35\end{cases}\\ &\begin{cases} \dfrac9{25}-6\cdot \dfrac35+10-a^2\leqslant 0\\[2ex] 3+a> \dfrac35\end{cases} \end{aligned}\end{gathered}\right.\] Решая данную совокупность и учитывая, что \(a<0\), получим: \[a\in \left(-\dfrac{13}5;-\dfrac{12}5\right]\]

Ответ:

\(\left(-\frac{13}5;-\frac{12}5\right] \cup\left[\frac{12}5;\frac{13}5\right)\)

Задание 10 #3264
Уровень задания: Равен ЕГЭ

Найдите все значения \(a\), при каждом из которых уравнение \[\ln (3x-1)\cdot \sqrt{x^2-8x+8a-a^2}=0\]

имеет ровно один корень на отрезке \([0;4]\).

 

(ЕГЭ 2017, основная волна)

Данное уравнение имеет следующие корни:
\(\bullet\) \(\ln (3x-1)=0\quad\Rightarrow\quad x=\frac23\) при \(\left(\frac23\right)^2-8\cdot \frac23+8a-a^2\geqslant 0\)   \(\bullet\) \(x^2-8x+8a-a^2=0\quad\Rightarrow\quad (x-a)(x+a-8)=0\quad\Rightarrow\quad\) \(x=a\) при \(3a-1>0 \ \) или \( \ x=8-a\) при \(23-3a>0\).   Случай I. Рассмотрим первый случай, когда уравнение имеет один корень на \([0;4]\). Пусть все корни различны, то есть \(a\ne \frac23; \ 4; \ \frac{22}3\).   1) Пусть этот корень \(x=\frac23\). Тогда, во-первых, \[\dfrac49-\dfrac{16}3+8a-a^2\geqslant 0\quad\Rightarrow\quad \dfrac23\leqslant a\leqslant \dfrac{22}3\] Учитывая, что корни не должны совпадать, получаем: \[a\in \left(\dfrac23; 4\right)\cup\left(4;\dfrac{22}3\right)\qquad (1)\] Во-вторых, корень \(x=a\) должен либо не удовлетворять ОДЗ \(3x-1>0\), либо не лежать на отрезке \([0;4]\): \[\left[\begin{gathered}\begin{aligned} &3a-1\leqslant 0\\ &a\notin [0;4]\end{aligned}\end{gathered}\right.\quad\Leftrightarrow\quad a\in \left(-\infty;\frac13\right]\cup(4;+\infty)\qquad (2)\] В-третьих, корень \(x=8-a\) также должен либо не удовлетворять ОДЗ \(3x-1>0\), либо не лежать на отрезке \([0;4]\): \[\left[\begin{gathered}\begin{aligned} &3(8-a)-1\leqslant 0\\ &8-a\notin [0;4]\end{aligned}\end{gathered}\right.\quad\Leftrightarrow\quad a\in \left(-\infty;4\right)\cup\left[\frac{23}3;+\infty\right)\qquad (3)\] Так как первое, второе и третье условие должны выполняться одновременно, то пересечем их решения и получим: \[a\in \varnothing\]   2) Пусть этот корень \(x=a\). Тогда, во-первых, \[\begin{cases} 3a-1>0\\ 0\leqslant a\leqslant 4\end{cases}\quad\Rightarrow\quad \dfrac13<a\leqslant 4\] Опять же, учитывая, что корни не должны совпадать, получаем \[a\in \left(\dfrac13;\dfrac23\right)\cup\left(\dfrac23;4\right)\] Во-вторых, заметим, что при этих \(a\) корень \(x=8-a\) всегда будет больше \(4\), то есть никогда не будет входить в отрезок \([0;4]\).
В-третьих, нужно, чтобы корень \(x=\frac23\) нам не подходил: \[\dfrac49-\dfrac{16}3+8a-a^2< 0\quad\Rightarrow\quad a\in \left(-\infty;\dfrac23\right)\cup\left(\dfrac{22}3;+\infty\right)\] Пересекая полученные значения для \(a\), получаем: \[a\in \left(\dfrac13;\dfrac23\right)\]   3) Пусть этот корень \(x=8-a\). Тогда, во-первых, \[\begin{cases} 3(8-a)-1>0\\ 0\leqslant 8-a\leqslant 4\end{cases}\quad\Rightarrow\quad 4\leqslant a<\dfrac{23}3\] Опять же, учитывая, что корни не должны совпадать, получаем \[a\in \left(4;\dfrac{22}3\right)\cup\left(\dfrac{22}3;\dfrac{23}3\right)\] Во-вторых, заметим, что при этих \(a\) корень \(x=a\) всегда будет больше \(4\), то есть никогда не будет входить в отрезок \([0;4]\).
В-третьих, нужно, чтобы корень \(x=\frac23\) нам не подходил: \[\dfrac49-\dfrac{16}3+8a-a^2< 0\quad\Rightarrow\quad a\in \left(-\infty;\dfrac23\right)\cup\left(\dfrac{22}3;+\infty\right)\] Пересекая полученные значения для \(a\), получаем: \[a\in \left(\dfrac{22}3;\dfrac{23}3\right)\]   Случай II. Рассмотрим случаи, когда какие-то два корня совпадают: когда \(a=\frac23\), или \(a=4\), или \(a=\frac{22}3\).
При \(a=\frac23\) первый и второй корень совпали, являются решением уравнения, а корень \(x=8-a\) не является. Следовательно, \(a=\frac23\) нам подходит.
При \(a=4\) совпадут второй и третий корни, они будут являться решением уравнения, а также корень \(x=\frac23\) будет являться решением. То есть уравнение будет иметь два корня, что нам не подходит.
При \(a=\frac{22}3\) совпадут первый и третий корни, решением уравнения они будут являться. Корень \(x=a\), очевидно, не будет лежать в отрезке \([0;4]\). Следовательно, уравнение будет иметь один корень, что нам подходит.
Итоговый ответ: \[a\in \left(\dfrac13;\dfrac23\right]\cup\left[\dfrac{22}3;\dfrac{23}3\right)\]

Ответ:

\(a\in \left(\frac13;\frac23\right]\cup\left[\frac{22}3;\frac{23}3\right)\)

Задание 11 #3218
Уровень задания: Равен ЕГЭ

Найдите все значения параметра \(a\), при каждом из которых уравнение \[\sqrt{x-a}\cdot \sin x=\sqrt{x-a}\cdot \cos x\]

имеет ровно один корень на отрезке \([0;\pi]\).

 

(ЕГЭ 2017, основная волна)

Преобразуем уравнение: \[\sqrt{x-a}\cdot (\sin x-\cos x)=0 \quad\Leftrightarrow\quad \begin{cases} \left[\begin{gathered}\begin{aligned} &x-a=0\\ &\sin x-\cos x=0 \end{aligned} \end{gathered}\right.\\ x-a\geqslant 0 \end{cases}\quad\Leftrightarrow\quad \begin{cases} \left[\begin{gathered}\begin{aligned} &x=a\\ &\mathrm{tg}\,x=1 \end{aligned} \end{gathered}\right.\\ x\geqslant a \end{cases} \quad\Leftrightarrow\quad \begin{cases} \left[\begin{gathered}\begin{aligned} &x_1=a\\ &x_2=\dfrac{\pi}4+\pi n, n\in\mathbb{Z} \end{aligned} \end{gathered}\right.\\ x\geqslant a \end{cases}\] Назовем решение неравенства \(x\geqslant a\) ОДЗ.
Заметим, что из серии корней \(x_2\) в отрезок \([0;\pi]\) попадает только корень \(x_2=\dfrac{\pi}4\). Следовательно, найдем, при каких значениях \(a\) система будет иметь одно решение на \([0;\pi]\): \[\begin{cases} \left[\begin{gathered}\begin{aligned} &x_1=a\\ &x_2=\dfrac{\pi}4 \end{aligned} \end{gathered}\right.\\ x\geqslant a \end{cases}\] Заметим, что если \(a>\pi\), то ОДЗ пересекается с отрезком \([0;\pi]\) по пустому множеству, следовательно, система не будет иметь ни одного решения на отрезке \([0;\pi]\). Значит, как минимум, \(a\leqslant \pi\). Рассмотрим три случая:

 

1) \(0<a\leqslant \pi\). Тогда ОДЗ: \(x\in [a;+\infty)\). ОДЗ в пересечении с отрезком \([0;\pi]\) дает отрезок \([a;\pi]\). Следовательно, нужно, чтобы система имела одно решение на \([a;\pi]\). Заметим, что в этом случае \(x_1=a\) всегда попадает в \([a;\pi]\). Значит, нужно, чтобы \(x_2=\frac{\pi}4\) не лежал на отрезке \([a;\pi]\) (то есть \(\frac{\pi}4<a\)), либо совпадал с точкой \(a\). Тогда система будет иметь на \([0;\pi]\) ровно один корень \(x_1=a\). Следовательно, обобщая все вышесказанное: \[\begin{cases} 0<a\leqslant \pi\\[1ex] \dfrac{\pi}4\leqslant a \end{cases} \quad\Leftrightarrow\quad \dfrac{\pi}4\leqslant a\leqslant \pi\]

2) Если \(a=0\), то ОДЗ: \(x\in [0;+\infty)\) и система имеет два корня \(x_1=0\) и \(x_2=\frac{\pi}4\) на \([0;\pi]\), следовательно, этот случай нам не подходит.

 

3) Пусть \(a<0\). Тогда ОДЗ: \(x\in [a;+\infty)\) и ОДЗ в пересечении с отрезком \([0;\pi]\) дает отрезок \([0;\pi]\). Тогда корень \(x_1=a\) не попадает в отрезок \([0;\pi]\), \(x_2=\frac{\pi}4\) попадает и система имеет на этом отрезке ровно одно решение.

 

Таким образом, искомые \(a\): \[a\in (-\infty;0)\cup\left[\dfrac{\pi}4;\pi\right]\]

Ответ:

\((-\infty;0)\cup\left[\dfrac{\pi}4;\pi\right]\)

Задание 12 #3230
Уровень задания: Равен ЕГЭ

Найдите все значения параметра \(a\), при каждом из которых система \[\begin{cases} |x|+|a|\leqslant 4\\ x^2+8x<16a+48 \end{cases}\]

имеет хотя бы одно решение на отрезке \([-1;0]\).

 

(ЕГЭ 2017, досрочная волна, резерв)

1 способ. Алгебраический

 

Заметим, что при \(|a|>4\) первое неравенство системы не будет иметь решений (так как тогда \(|x|\) должен быть не больше отрицательного числа), следовательно, и вся система не будет иметь решений.
При \(|a|=4\) решением первого неравенства будет \(x=0\) (\(\in [-1;0]\)). Заметим, что пара \(x=0\) и \(a=4\) является решением второго неравенства, пара \(x=0\) и \(a=-4\) – нет.
Следовательно, \(a=4\) – подходит.

 

1) Пусть \(0<a<4\). Тогда \(|a|=a\) и система перепишется в виде: \[\begin{cases} |x|\leqslant 4-a\\ (x+4)^2<16(a+4)\end{cases} \quad\Rightarrow\quad \begin{cases} -4+a\leqslant x\leqslant 4-a\\ -4-4\sqrt{4+a}<x<-4+4\sqrt{4+a} \end{cases}\] Заметим, что \(-4+a<0; \ -4+4\sqrt{4+a}>4>4-a\), \(-4-4\sqrt{4+a}<-4+a\), следовательно:



Таким образом, решением системы будут \(x\in [-4+a;4-a]\), что содержит хотя бы одну точку из \([-1;0]\) (например, \(x=0\)).
Таким образом, все \(a\in (0;4)\) – подходят.

 

2) При \(a=0\) система перепишется в виде: \[\begin{cases} -4\leqslant x\leqslant 4\\ -12<x<4 \end{cases} \quad\Leftrightarrow\quad -4\leqslant x<4\] Следовательно, \(a=0\) – подходит.

 

3) Пусть \(-4<a<0\). Тогда \(|a|=-a\) и система примет вид: \[\begin{cases} |x|\leqslant 4+a\\ (x+4)^2<16(4+a) \end{cases} \quad\Rightarrow\quad \begin{cases} -4-a\leqslant x\leqslant 4+a\\ -4-4\sqrt{4+a}<x<-4+4\sqrt{4+a} \end{cases}\] Заметим, что \(-4-a>-4\), а \(-4-4\sqrt{4+a}<-4\). Также \(-4+4\sqrt{4+a}\leqslant 4+a\) (так как \((4+a)-4\sqrt{4+a}+4\geqslant 0 \ \Rightarrow \ (\sqrt{4+a}-2)^2\geqslant 0\) – верно при всех \(a\in(-4;0)\))
Следовательно, нам подходит такая картинка:



То есть \(-4+4\sqrt{4+a}\) должно быть больше \(-4-a\) (тогда решением будут \(x\in [-4-a;-4+4\sqrt{4+a})\)), а также больше \(-1\) (тогда хотя бы одно число из \([-1;0]\) будет содержаться в решении). \[\begin{cases} -4+4\sqrt{4+a}>-4-a\\ -4+4\sqrt{4+a}>-1 \end{cases}\quad\Rightarrow\quad \begin{cases} 8-8\sqrt2<a<0\\[1ex] a>-\dfrac{55}{16} \end{cases}\] (так как \(a\in (-4;0)\)).
Для того, чтобы дать окончательный ответ, нужно сравнить числа \(8-8\sqrt2\) и \(-\frac{55}{16}\): \[\begin{aligned} 8\sqrt2-8 &\lor \dfrac{55}{16}\\[1ex] 128\sqrt2 &\lor 183\\ 32768 &\lor 33489\end{aligned}\] Следовательно, \(8\sqrt2-8<\dfrac{55}{16}\), значит, \(8-8\sqrt2>-\dfrac{55}{16}\), следовательно, \(a\in (8-8\sqrt2;0)\).

 

Тогда окончательный ответ для \(a\): \[a\in (8-8\sqrt2;4]\]

 

2 способ. Геометрический

 

Рассмотрим прямоугольную систему координат \(xOa\), где ось \(Oa\) — ось ординат.
Изобразим область, являющуюся решением системы. Тогда те значения \(a\), при которых существуют точки \((x;a)\) из этой области с \(x\in [-1;0]\), пойдут в ответ.

 

Второе неравенство системы можно переписать в виде \[a>\dfrac1{16}(x+4)^2-4\] Следовательно, оно задает область между ветвями параболы \(a=\frac1{16}(x+4)^2-4\) (ветви параболы направлены вверх). Вершина параболы находится в точке \((-4;-4)\), пересекает ось абсцисс парабола в точках \(x=-12\) и \(x=4\) (находятся из уравнения \(0=\frac1{16}(x+4)^2-4\)).


 

Изобразим область, являющуюся решением первого неравенства. При \(x,a\) из \(I\) четверти (то есть \(x>0, a>0\)) неравенство переписывается в виде \(x+a\leqslant 4\quad\Rightarrow\quad a\leqslant 4-x\) и задает область первой четверти, находящуюся под прямой. Аналогично рассмотрев по отдельности случаи, когда \(x, a\) лежат в \(II\), \(III\) и \(IV\) четвертях, найдем, что первое неравенство задает внутренность квадрата:



Тогда зеленая область – область, являющаяся решением системы.
Найдем все \(x\in[-1;0]\) и лежащие в это области. Для этого необходимо найти точку \((x_0;a_0)\) пересечения параболы со стороной квадрата из \(III\) четверти и понять, правее или левее \(-1\) ее абсцисса \(x_0\) (заметим, что эта точка не будет лежать в области, так как граница области, являющаяся частью параболы, не входит в область).
Сторона квадрата в \(III\) четверти задается уравнением \(-x-a=4\), откуда \(a=-(x+4)\). Следовательно, \[-(x+4)=\dfrac1{16}(x+4)^2-4 \quad\Leftrightarrow\quad (x+12)^2=2\cdot 64 \quad\Leftrightarrow\quad x=-12\pm 8\sqrt2\] Так как \(x_0\in (-4;0)\), то подходит \(x_0=-12+8\sqrt2\). Сравним \(-12+8\sqrt2\) и \(-1\): \[\begin{aligned} 12-8\sqrt2 &\lor 1\\ 11 &\lor 8\sqrt2\\ 121 &\lor 128\end{aligned}\] Следовательно, \(12-8\sqrt2<1\), значит, \(-12+8\sqrt2>-1\).



Таким образом, мы видим, что точки из области, лежащие в \([-1;0]\), имеют ординаты \(a\): \[a_0<a\leqslant 4 \quad\Rightarrow\quad -(-12+8\sqrt2+4)<a\leqslant 4 \quad\Rightarrow\quad 8-8\sqrt2<a\leqslant 4\]

Ответ:

\((8-8\sqrt2;4]\)

Задание 13 #2974
Уровень задания: Равен ЕГЭ

Найдите все значения параметра \(a\), при каждом из которых система \[\begin{cases} ax\geqslant 2\\ 3x\leqslant 2a+11\\ \sqrt{x-1}>a \end{cases}\]

имеет хотя бы одно решение, принадлежащее отрезку \([3;4]\).

 

(ЕГЭ 2017, досрочная волна)

1 способ.

 

1) Рассмотрим случай, когда \(a>0\). В этом случае систему можно переписать в виде: \[\begin{cases} x\geqslant \dfrac2a\\[2ex] x\leqslant \dfrac{2a+11}3\\[2ex] x>a^2+1 \end{cases} \qquad (*)\]

Для того, чтобы система имела решения, нужно, чтобы \[\begin{cases} \dfrac2a\leqslant \dfrac{2a+11}3\\[3ex] a^2+1<\dfrac{2a+11}3\end{cases}\]

Решением неравенства \(\dfrac2a\leqslant \dfrac{2a+11}3\) будут \(a\in (-\infty;-6]\cup\left[\frac12;+\infty\right)\). Так как \(a>0\), то подходит только \(a\geqslant \dfrac12\).

 

Решением неравенства \(a^2+1<\dfrac{2a+11}3\) будут \(a\in (0;2)\). Следовательно, пересекая полученные решения, имеем: \(a\in \left[\frac12;2\right)\). Таким образом, при этих \(a\) система \((*)\) будет иметь решения.

 

Теперь посмотрим, когда хотя бы одно из этих решений будет лежать в отрезке \([3;4]\).

Заметим, что при полученных \(a\) числа \(\dfrac2a; \ a^2+1; \ \dfrac{2a+11}3\) могут располагаться в следующем порядке: \[\begin{aligned} & I. \quad \dfrac2a; \ a^2+1; \ \dfrac{2a+11}3 \quad \Rightarrow\quad {\small{\text{тогда решением системы (*) будут }}} x\in \left(a^2+1;\dfrac{2a+11}3\right]\\[4ex] & II. \quad \dfrac2a=a^2+1; \ \dfrac{2a+11}3 \quad \Rightarrow\quad {\small{\text{тогда решением системы (*) будут }}} x\in \left(a^2+1;\dfrac{2a+11}3\right]\\[4ex] & III. \quad a^2+1; \ \dfrac2a; \ \dfrac{2a+11}3 \quad \Rightarrow\quad {\small{\text{тогда решением системы (*) будут }}} x\in \left[\dfrac2a;\dfrac{2a+11}3\right] \end{aligned}\]

\(I\) и \(II\) случаи задаются условием \(\dfrac2a\leqslant a^2+1\).
В этих случаях для того, чтобы хотя бы одно решение попало в отрезок \([3;4]\), нужно, чтобы \(a^2+1<4\).
Следовательно, решим систему: \[\begin{cases} \dfrac2a\leqslant a^2+1\\[2ex] a^2+1<4 \end{cases} \quad\Rightarrow\quad \begin{cases} a^3+a-2\geqslant 0\\ a^2<3 \end{cases}\quad\Rightarrow\quad \begin{cases} (a-1)(a^2+a+2)\geqslant 0\\ a^2<3 \end{cases}\] Следовательно, учитывая, что \(a\in \left[\frac12;2\right)\), решением системы будут: \(a\in [1;\sqrt3)\).

Случай \(III\) задается условием \(a^2+1<\dfrac2a\).
В этом случае для того, чтобы хотя бы одно решение попало в отрезок \([3;4]\), нужно, чтобы \(\dfrac2a\leqslant 4\).
Следовательно, решим систему: \[\begin{cases} a^2+1<\dfrac2a\\[2ex] \dfrac2a\leqslant 4 \end{cases} \quad\Rightarrow\quad \begin{cases} a^3+a-2<0\\ a\geqslant \dfrac12 \end{cases}\] Следовательно, учитывая, что \(a\in \left[\frac12;2\right)\), решением системы будут: \(a\in \left[\frac12;1\right)\).

Так как нам подходит или случай \(I\), или \(II\), или \(III\), то значения \(a\), полученные в этих случаях, нужно объединить. Объединяя \(\left[\frac12;1\right)\) и \([1;\sqrt3)\), получим \[a\in \left[\dfrac12;\sqrt3\right).\]

 

2 способ.

 

Так как нужно, чтобы система имела хотя бы одно решение из отрезка \([3;4]\), то как минимум \(x>0\). Следовательно, решим систему только для \(x>0\). В таком случае можно разделить первое неравенство на \(x\) и получим следующую систему: \[\begin{cases} a\geqslant \dfrac2x\\[3ex] a\geqslant \dfrac32x-\dfrac{11}2\\[3ex] a<\sqrt{x-1} \end{cases}\]

Рассмотрим систему координат \(xOa\) (то есть привычная нам ось \(Oy\) будет называться \(Oa\)). Тогда каждое неравенство при \(x>0\) задает некоторую область, а решением системы является область, равная пересечению всех трех областей, как показано на рисунке (белая область и есть решение системы):


 

Тогда условие “система имеет хотя бы одно решение из отрезка \([3;4]\)” задает область:


 

Следовательно, \(a\in \left[a_1;a_2\right)\), где \(a_1, a_2\) – ординаты точек пересечения прямой \(x=4\) с \(a=\dfrac2x\) и \(x=4\) с \(a=\sqrt{x-1}\).
(Заметим, что значение \(a_1\) включается, так как неравенство \(a\geqslant \dfrac2x\) не строгое, а \(a_2\) не включается, так как неравенство \(a<\sqrt{x-1}\) строгое.)

 

Таким образом, \(a_1=\dfrac24=\dfrac12\), а \(a_2=\sqrt{4-1}=\sqrt3\). Следовательно, ответ \(a\in \left[\dfrac12;\sqrt3\right).\)

Ответ:

\(\left[\dfrac12;\sqrt3\right)\)

Задание 14 #2732
Уровень задания: Равен ЕГЭ

Найдите все значения параметра \(a\), при каждом из которых система уравнений \[\begin{cases} \begin{aligned} &(x-3)(y+3x-9)=|x-3|^3\\ &y=x+a \end{aligned} \end{cases}\]

имеет ровно четыре различных решения.

 

(ЕГЭ 2016, резерв)

Система равносильна уравнению: \((x-3)(4x+a-9)=|x-3|^3\), что равносильно: \[\left[ \begin{gathered} \begin{aligned} &\begin{cases} \begin{aligned} &(x-3)(4x+a-9-(x-3)^2)=0\\ &x-3 \geqslant 0 \end{aligned} \end{cases}\\ &\begin{cases} \begin{aligned} &(x-3)(4x+a-9+(x-3)^2)=0\\ &x-3<0 \end{aligned} \end{cases} \end{aligned} \end{gathered} \right. \Leftrightarrow \left[ \begin{gathered} \begin{aligned} &\begin{cases} \begin{aligned} &(x-3)(x^2-10x+18-a)=0\\ &x\geqslant 3 \end{aligned} \end{cases} \text{(1)}\\ &\begin{cases} \begin{aligned} &(x-3)(x^2-2x+a)=0\\ &x<3 \end{aligned} \end{cases} \text{(2)} \end{aligned} \end{gathered} \right.\]

Данная система будет иметь 4 решения в одном из следующих случаев:

 

1) Система (1) имеет 3 решения, а система (2) – 1 решение.

 

Для того, чтобы (1) имела 3 решения, необходимо, чтобы уравнение \(x^2-10x+18-a=0\) имело два корня и оба корня удовлетворяли условию \(x> 3\) (ни один из корней не может быть равен 3, потому что в этом случае система будет иметь 2 различных решения, а не 3).

 

Обозначим \(f(x)=x^2-10x+18-a\). Тогда необходимо, чтобы: \[\begin{cases} \begin{aligned} & D=4(7+a)>0\\ & \dfrac{10}{2}>3\\ & f(3)>0 \end{aligned} \end{cases}\]

(условие \(\frac{10}2>3\) означает, что вершина параболы находится правее \(3\))
Решив данную систему, получим \(a\in (-7;-3)\).

 

Для того, чтобы (2) имела 1 решение, необходимо выполнение одного из двух условий:

 

а) либо уравнение \(x^2-2x+a=0\) имеет 1 корень и он меньше 3, т.е. \[\begin{cases} \begin{aligned} & D=4(1-a)=0\\ & \dfrac{2}{2}<3 \end{aligned} \end{cases}\]

(условие \(\frac22>3\) означает, что вершина параболы находится левее \(3\))
Следовательно, \(a=1\).

 

б) либо уравнение \(x^2-2x+a=0\) имеет 2 корня (обозначим \(g(x)=x^2-2x+a\)) и эти корни находятся по разные стороны от 3, т.е.: \[\begin{cases} \begin{aligned} & D=4(1-a)>0\\ & g(3)<0 \end{aligned} \end{cases}\]

Следовательно, \(a<-3\).

 

Пересекая \(a\in (-7;-3)\) с \(a\in (-\infty;-3)\cup \{1\}\), получим \(a\in (-7;-3)\).

 

2) Система (1) имеет 2 решения и система (2) – 2 решения.

 

Для того, чтобы (1) имела 2 решения, необходимо, чтобы либо уравнение \(x^2-10x+18-a=0\) имело 1 корень, либо чтобы оно имело 2 корня, но только один из них был бы больше 3. Следовательно, \(a\in \{-7\}\cup(-3;+\infty)\).

 

Для того, чтобы (2) имела 2 решения, необходимо, чтобы уравнение \(x^2-2x+a=0\) имело 2 корня, причем оба корня были меньше 3. Следовательно, \(a\in (-3;1)\).

 

Пересекая \(a\in \{-7\} \cup(-3;+\infty)\) с \(a\in (-3;1)\), получим \(a\in (-3;1)\).

Ответ:

\(a\in (-7;-3)\cup (-3;1)\).